Difference between revisions of "2000 AMC 10 Problems/Problem 17"

(See Also)
m (Problem)
Line 3: Line 3:
 
Boris has an incredible coin changing machine. When he puts in a quarter, it returns five nickels; when he puts in a nickel, it returns five pennies; and when he puts in a penny, it returns five quarters. Boris starts with just one penny. Which of the following amounts could Boris have after using the machine repeatedly?
 
Boris has an incredible coin changing machine. When he puts in a quarter, it returns five nickels; when he puts in a nickel, it returns five pennies; and when he puts in a penny, it returns five quarters. Boris starts with just one penny. Which of the following amounts could Boris have after using the machine repeatedly?
  
<math>\mathrm{(A)}</math> <dollar/><math>3.63</math>  
+
<math>\mathrm{(A)}</math> <math>\$3.63</math>  
  
<math>\mathrm{(B)}</math> <dollar/><math>5.13</math>
+
<math>\mathrm{(B)}</math> <math>\$5.13</math>
  
<math>\mathrm{(C)}</math> <dollar/><math>6.30</math>  
+
<math>\mathrm{(C)}</math> <math>\$6.30</math>  
  
<math>\mathrm{(D)}</math> <dollar/><math>7.45</math>  
+
<math>\mathrm{(D)}</math> <math>\$7.45</math>  
  
<math>\mathrm{(E)}</math> <dollar/><math>9.07</math>
+
<math>\mathrm{(E)}</math> <math>\$9.07</math>
  
 
==Solution==
 
==Solution==

Revision as of 23:40, 18 November 2015

Problem

Boris has an incredible coin changing machine. When he puts in a quarter, it returns five nickels; when he puts in a nickel, it returns five pennies; and when he puts in a penny, it returns five quarters. Boris starts with just one penny. Which of the following amounts could Boris have after using the machine repeatedly?

$\mathrm{(A)}$ $$3.63$

$\mathrm{(B)}$ $$5.13$

$\mathrm{(C)}$ $$6.30$

$\mathrm{(D)}$ $$7.45$

$\mathrm{(E)}$ $$9.07$

Solution

Consider what happens each time he puts a coin in. If he puts in a quarter, he gets five nickels back, so the amount of money he has doesn't change. Similarly, if he puts a nickel in the machine, he gets five pennies back and the money value doesn't change. However, if he puts a penny in, he gets five quarters back, increasing the amount of money he has by $124$ cents.

This implies that the only possible values, in cents, he can have are the ones one more than a multiple of $124$. Of the choices given, the only one is $\boxed{\text{D}}$

See Also

2000 AMC 10 (ProblemsAnswer KeyResources)
Preceded by
Problem 16
Followed by
Problem 18
1 2 3 4 5 6 7 8 9 10 11 12 13 14 15 16 17 18 19 20 21 22 23 24 25
All AMC 10 Problems and Solutions

The problems on this page are copyrighted by the Mathematical Association of America's American Mathematics Competitions. AMC logo.png